Théorème des valeurs intermédiaires

Exercices du dossier Théorème des valeurs intermédiaires

Exercice 308 *

13 janvier 2021 20:48 — Par Alain Soyeur François Capaces Emmanuel Vieillard-Baron

Soit \(f\) une fonction polynomiale de degré impair. Montrer que \(f\) possède au moins une racine réelle.



[ID: 632] [Date de publication: 13 janvier 2021 20:48] [Catégorie(s): Théorème des valeurs intermédiaires ] [ Nombre commentaires: 0] [nombre d'éditeurs: 1 ] [Editeur(s): Emmanuel Vieillard-Baron ] [nombre d'auteurs: 3 ] [Auteur(s): Alain Soyeur François Capaces Emmanuel Vieillard-Baron ]
Accordéon
Titre
Solution
Texte

Exercice 308
Par Alain Soyeur François Capaces Emmanuel Vieillard-Baron le 13 janvier 2021 20:48

Supposons que le coefficient du terme dominant de \(P\) soit positif. Comme \(P\) est de degré impair, on en déduit que \(\displaystyle{\lim_{x \rightarrow -\infty}f\left(x\right)}=-\infty\) et que \(\displaystyle{\lim_{x \rightarrow +\infty}f\left(x\right)}=+\infty\). Comme \(f\) est polynomiale, elle est continue et donc, par application du théorème des valeurs intermédiaires \(f\left(\mathbb{R}\right)=\mathbb{R}\). Il existe donc \(c\in\mathbb{R}\) tel que \(\boxed{f\left(c\right)=0}\). On raisonne de même si le coefficient du terme dominant de \(P\) est négatif.


Exercice 717 *

13 janvier 2021 20:49 — Par Alain Soyeur François Capaces Emmanuel Vieillard-Baron

Soit \(f:\mathbb{R}\rightarrow \mathbb{R}\) continue telle que \(\displaystyle{\lim_{x \to -\infty} f(x) = -1}\) et \(\displaystyle{\lim_{x \to +\infty} f(x) = +1}\). Prouver que \(f\) s’annule



[ID: 634] [Date de publication: 13 janvier 2021 20:49] [Catégorie(s): Théorème des valeurs intermédiaires ] [ Nombre commentaires: 0] [nombre d'éditeurs: 1 ] [Editeur(s): Emmanuel Vieillard-Baron ] [nombre d'auteurs: 3 ] [Auteur(s): Alain Soyeur François Capaces Emmanuel Vieillard-Baron ]
Accordéon
Titre
Solution
Texte

Exercice 717
Par Alain Soyeur François Capaces Emmanuel Vieillard-Baron le 13 janvier 2021 20:49

Comme \(\displaystyle{\lim_{x \rightarrow -\infty}f\left(x\right)}=-1\), il existe \(a\in\mathbb{R}\) tel que \(f\left(a\right)<0\). De même, comme \(\displaystyle{\lim_{x \rightarrow +\infty}f\left(x\right)}=+1\), il existe \(b\in\mathbb{R}\) tel que \(f\left(b\right)>0\). \(f\) étant continue, d’après le théorème des valeurs intermédiaires appliqué sur le segment \(\left[a,b\right]\), on en déduit qu’il existe \(c\in\left[a,b\right]\) tel que \(f\left(c\right)=0\).


Un théorème classique de point fixe *

13 janvier 2021 20:49 — Par Alain Soyeur François Capaces Emmanuel Vieillard-Baron

Soit \(f:[0,1] \rightarrow [0,1]\) continue. Prouver que \(f\) possède au moins un point fixe.



[ID: 636] [Date de publication: 13 janvier 2021 20:49] [Catégorie(s): Théorème des valeurs intermédiaires ] [ Nombre commentaires: 0] [nombre d'éditeurs: 1 ] [Editeur(s): Emmanuel Vieillard-Baron ] [nombre d'auteurs: 3 ] [Auteur(s): Alain Soyeur François Capaces Emmanuel Vieillard-Baron ]
Accordéon
Titre
Solution
Texte

Un théorème classique de point fixe
Par Alain Soyeur François Capaces Emmanuel Vieillard-Baron le 13 janvier 2021 20:49

Introduisons la fonction \(g\) donnée par \(\forall x \in [0,1], \quad g(x)=f(x)-x\). La fonction \(g\) est continue sur \([0,1]\), \(g(0)=f(0) \geqslant 0\) et \(g(1)=f(1)-1 \leqslant 0\) donc, d’après le théorème des valeurs intermédiaires, \(g\) s’annule sur \([0,1]\).


Exercice 418 *

13 janvier 2021 20:49 — Par Alain Soyeur François Capaces Emmanuel Vieillard-Baron

Montrer que les seules applications continues de \(\mathbb{R}\) dans \(\mathbb{Z}\) sont les applications constantes.



[ID: 638] [Date de publication: 13 janvier 2021 20:49] [Catégorie(s): Théorème des valeurs intermédiaires ] [ Nombre commentaires: 0] [nombre d'éditeurs: 1 ] [Editeur(s): Emmanuel Vieillard-Baron ] [nombre d'auteurs: 3 ] [Auteur(s): Alain Soyeur François Capaces Emmanuel Vieillard-Baron ]
Accordéon
Titre
Solution
Texte

Exercice 418
Par Alain Soyeur François Capaces Emmanuel Vieillard-Baron le 13 janvier 2021 20:49

Soit \(f:\mathbb{R}\rightarrow \mathbb{Z}\), Suposons que \(f\) n’est pas constante. Il existe alors des réels \(a<b\) tels que \(f(a) \neq f(b)\). Soit \(y\) un nombre non entier strictement compris entre \(f(a)\) et \(f(b)\). D’après le théorème des valeurs intermédiaires, il existe \(x\in]a,b[\) tel que \(f(x)=y\) et donc \(f\) n’est pas à valeurs dans \(\mathbb{Z}\). \(f\) est donc forcément constante.


Exercice 642 **

13 janvier 2021 20:49 — Par Alain Soyeur François Capaces Emmanuel Vieillard-Baron

On considère un méridien terrestre et l’on suppose que la température au sol varie continument sur ce méridien. Montrez l’existence de deux points antipodaux sur ce méridien où la température est la même.



[ID: 640] [Date de publication: 13 janvier 2021 20:49] [Catégorie(s): Théorème des valeurs intermédiaires ] [ Nombre commentaires: 0] [nombre d'éditeurs: 1 ] [Editeur(s): Emmanuel Vieillard-Baron ] [nombre d'auteurs: 3 ] [Auteur(s): Alain Soyeur François Capaces Emmanuel Vieillard-Baron ]
Accordéon
Titre
Solution
Texte

Exercice 642
Par Alain Soyeur François Capaces Emmanuel Vieillard-Baron le 13 janvier 2021 20:49

À chaque point du méridien, on associe l’angle \(\theta\) entre le pôle nord et ce point. Considérons la fonction \(f : [0, 2\pi] \mapsto \mathbb{R}\)\(f(\theta)\) représente la température au point d’angle \(\theta\). Par hypothèse, cette fonction est continue et \(f(0) = f(2\pi) = T\)\(T\) est la température au pôle nord. Considérons la fonction définie sur \([0, \pi]\) par \(g(\theta) = f(\theta+\pi) - f(\theta)\). Comme \(g\) est continue et que \(g(0) = f(\pi) - f(0) = f(\pi) - f(2\pi) = - g(\pi)\), d’après le théorème des valeurs intermédiaires, il existe \(\alpha \in ]0, \pi[\) tel que \(g(\alpha) = 0\), c’est-à-dire \(f(\alpha + \pi) = f(\alpha)\).


Exercice 251 **

13 janvier 2021 20:49 — Par Alain Soyeur François Capaces Emmanuel Vieillard-Baron

Soit \(f:\mathbb{R}\rightarrow \mathbb{R}\) continue et décroissante. Montrer que \(f\) admet un unique point fixe.



[ID: 642] [Date de publication: 13 janvier 2021 20:49] [Catégorie(s): Théorème des valeurs intermédiaires ] [ Nombre commentaires: 0] [nombre d'éditeurs: 1 ] [Editeur(s): Emmanuel Vieillard-Baron ] [nombre d'auteurs: 3 ] [Auteur(s): Alain Soyeur François Capaces Emmanuel Vieillard-Baron ]
Accordéon
Titre
Solution
Texte

Exercice 251
Par Alain Soyeur François Capaces Emmanuel Vieillard-Baron le 13 janvier 2021 20:49

Introduisons la fonction \(g\) donnée par \(\forall x\in \mathbb{R}, \quad g(x)=f(x)-x\). \(g\) est strictement décroissante et donc injective et ne s’annule donc qu’une fois au plus. Supposons que \(g\) ne s’annule pas. Alors \(g\) est ou strictement positive ou strictement négative.

  1. Si \(g>0\) alors \(\forall x \in \mathbb{R}, \quad f(x) >x\) et donc \(f(x) \xrightarrow[x\rightarrow +\infty]{} +\infty\) ce qui est absurde car \(\displaystyle{\lim_{+\infty} f = \inf_\mathbb{R}f}\).

  2. Si \(g<0\) alors \(\forall x \in \mathbb{R}, \quad f(x) <x\) et donc \(f(x) \xrightarrow[x\rightarrow -\infty]{} -\infty\) ce qui est absurde car \(\displaystyle{\lim_{-\infty} f = \sup_\mathbb{R}f}\).

On aboutit dans les deux cas à une contradiction et nécessairement \(g\) s’annule une et une seule fois sur \(\mathbb{R}\). On en déduit que \(f\) admet un et un seul point fixe.


Exercice 835 **

13 janvier 2021 20:49 — Par Alain Soyeur François Capaces Emmanuel Vieillard-Baron

Soit une fonction \(f:[0,1]\mapsto \mathbb{R}\) continue sur le segment \([0,1]\). Soient deux réels \(p,q >0\). Montrer qu’il existe \(x_0\in [0,1]\) tel que \[pf(0)+qf(1)=(p+q)f(x_0).\]



[ID: 644] [Date de publication: 13 janvier 2021 20:49] [Catégorie(s): Théorème des valeurs intermédiaires ] [ Nombre commentaires: 0] [nombre d'éditeurs: 1 ] [Editeur(s): Emmanuel Vieillard-Baron ] [nombre d'auteurs: 3 ] [Auteur(s): Alain Soyeur François Capaces Emmanuel Vieillard-Baron ]
Accordéon
Titre
Solution
Texte

Exercice 835
Par Emmanuel Vieillard-Baron le 13 janvier 2021 20:52

Introduisons la fonction définie par \(\varphi(x)=(p+q)f(x)-pf(0)-qf(1)\).

Cette fonction \(\varphi\) est continue sur le segment \([0,1]\) et \[\varphi(0)=q(f(0)-f(1)), \quad\varphi(1)=p(f(1)-f(0))\] Comme \(p,q>0\), \(\varphi(0)\) et \(\varphi(1)\) sont de signes opposés. D’après le théorème des valeurs intermédiaires, il existe \(x_0\in [0,1]\) tel que \(\varphi(x_0)=0\), ce qui prouve le résultat.


Exercice 474 **

13 janvier 2021 20:49 — Par Alain Soyeur François Capaces Emmanuel Vieillard-Baron

Soit une fonction \(f:\mathbb{R} \mapsto \mathbb{R}\) continue et croissante. On suppose qu’il existe un réel \(a>0\) tel que \[\forall (x,y)\in \mathbb{R}^{2}, \quad\left| f(x)-f(y) \right| \geqslant a\left| x-y \right|\] Montrer que la fonction \(f\) est bijective.



[ID: 646] [Date de publication: 13 janvier 2021 20:49] [Catégorie(s): Théorème des valeurs intermédiaires ] [ Nombre commentaires: 0] [nombre d'éditeurs: 1 ] [Editeur(s): Emmanuel Vieillard-Baron ] [nombre d'auteurs: 3 ] [Auteur(s): Alain Soyeur François Capaces Emmanuel Vieillard-Baron ]
Accordéon
Titre
Solution
Texte

Exercice 474
Par Alain Soyeur François Capaces Emmanuel Vieillard-Baron le 13 janvier 2021 20:49
  1. Montrons que \(f\) est injective : soient deux réels \((x,y)\in \mathbb{R}^{2}\) tels que \(f(x)=f(y)\). Alors \[\lvert x-y \rvert \leqslant\dfrac{1}{a}\left| f(x)-f(y) \right| \leqslant 0\] et donc \(x=y\).

  2. Montrons que la fonction \(f\) n’est pas majorée. Par l’absurde : si \(f\) était majorée alors d’après le théorème de la limite monotone, elle tendrait vers une limite finie \(l\) lorsque \(x\rightarrow +\infty\). Mais alors, il existerait \(c>0\) tel que \(\forall x\geqslant c\), \(l-1 \leqslant f(x) \leqslant l\). On aurait alors , \[\forall x\geqslant c,\quad \lvert f(x)-f(c) \rvert \geqslant a\lvert x-c \rvert \Rightarrow \lvert x-c \rvert \leqslant\dfrac{1}{a} \lvert f(x)-f(c) \rvert \leqslant\dfrac{1}{a}\] ce qui est impossible car pour \(x\) assez grand, \(\lvert x-c \rvert > \dfrac{1}{a}\). On montre de même que \(f\) n’est pas minorée.

  3. Par conséquent, la fonction \(f\) est surjective. En effet, Soit \(t\in \mathbb{R}\). Comme \(\lim_{x\rightarrow +\infty}f(x)=+\infty\) et \(\lim_{x\rightarrow -\infty}f(x)=-\infty\), il existe \((a,b)\in \mathbb{R}^{2}\) tels que \(f(a)\leqslant t \leqslant f(b)\). Mais alors d’après le théorème des valeurs intermédiaires, \(\exists c\in [a,b]\) tel que \(f(c)=t\).


Exercice 653 **

13 janvier 2021 20:49 — Par Alain Soyeur François Capaces Emmanuel Vieillard-Baron

Soit \(f:[0,1]\mapsto [0,1]\) une fonction continue.

  1. Montrer que \(\forall n\in \mathbb{N}^{*}\), il existe \(a_n\in [0,1]\) tel que \(f(a_n)=a_n^n\) ;

  2. On suppose maintenant que \(f\) est décroissante strictement. Montrer que pour tout \(n\in\mathbb{N}^*\), le réel \(a_n\in [0,1]\) trouvé dans la question précédente est unique à vérifier \(f(a_n)=a_n^n\) et étudier la suite \((a_n)\).



[ID: 648] [Date de publication: 13 janvier 2021 20:49] [Catégorie(s): Théorème des valeurs intermédiaires ] [ Nombre commentaires: 0] [nombre d'éditeurs: 1 ] [Editeur(s): Emmanuel Vieillard-Baron ] [nombre d'auteurs: 3 ] [Auteur(s): Alain Soyeur François Capaces Emmanuel Vieillard-Baron ]
Accordéon
Titre
Solution
Texte

Exercice 653
Par Alain Soyeur François Capaces Emmanuel Vieillard-Baron le 13 janvier 2021 20:49
  1. Soit \(n>0\). Posons \(g_n:\left\{ \begin{array}{ccl} [0,1] & \longrightarrow & \mathbb{R} \\ x & \longmapsto & f(x)-x^n \end{array} \right.\). Alors la fonction \(g_n\) est continue sur \([0,1]\) et \(g_n(0)=f(0)\geqslant 0\), \(g_n(1)=f(1)-1 \leqslant 0\). D’après le théorème des valeurs intermédiaires, il existe \(a_n\in [0,1]\) tel que \(g_n(a_n)=0\) et donc \(f(a_n)=a_n^n\).

  2. Si \(f\) est continue et strictement décroissante, alors pour tout \(n\in\mathbb{N}^*\), \(g_n\) est continue et strictement décroissante également. Par conséquent, \(g_n\) réalise une bijection de \([0,1]\) vers \([f(1)-1,f(0)]\). Comme \(0\in [f(1)-1,f(0)]\), \(0\) possède un unique antécédent \(a_n\) par \(g_n\). Calculons \(g_{n+1}(a_n)=f(a_n)-a_n^{n+1}=a_n^n - a_n^{n+1}=a_n^n(1-a_n) \geqslant 0\) (car \(g_n(a_n)=0 \Rightarrow f(a_n)=a_n^n\)). Comme \(g_{n+1}\) est décroissante, \(a_{n}\leqslant a_{n+1}\) (par l’absurde, si \(a_{n+1}<a_n\), on aurait \(0=g_{n+1}(a_{n+1})> g_{n+1}(a_n)\geqslant 0\)). (Un petit coup d’œil sur le tableau de variations "évite" un raisonnement par l’absurde).

    La suite \((a_n)\) est croissante et majorée par \(1\), elle converge donc d’après le théorème de la limite monotone vers un réel \(l\in [0,1]\).


Accordéon
Titre
Solution
Texte

Exercice 162
Par Alain Soyeur François Capaces Emmanuel Vieillard-Baron le 13 janvier 2021 20:49
  1. Soit \(x \in K\). Montrons que \(g(x) \in K\). Calculons pour cela \(f(g(x)) = g(f(x)) = g(x)\).

  2. \(K\) est non vide, voir l’exercice et minoré par \(0\) donc \(K\) possède une borne inférieure \(x_0\). Montrons que \(x_0 \in K\). En appliquant la propriété de caractérisation de la borne inférieure, on construit une suite \(\left(x_n\right)\) de points de \(K\) qui converge vers \(x_0\). Comme \(\forall n \in \mathbb N\), \(f(x_n) = x_n\), et que \(f\) est continue au point \(x_0\), il vient en passant à la limite que \(f(x_0) = x_0\), donc \(x_0 \in K\).

  3. Soit \(n \in \mathbb N\). On a \(x_{n+1} = g(x_n) \geqslant x_n\) et donc la suite \((x_n)\) est croissante majorée par \(1\). Elle converge d’après le théorème de la limite monotone vers \(\ell \in [0, 1]\). Comme \(\forall n \in \mathbb N\), \(f(x_n) = x_n\) ( démonstration facile par récurrence), et que \(f\) est continue au point \(\ell\), on trouve que \(f(\ell) = \ell\). Comme également \(\forall n \in \mathbb N\), \(x_{n+1} = g(x_n)\), on a aussi \(g(\ell) = \ell\) et donc \(\ell\) est un point fixe commun à \(f\) et \(g\).

Remarque : On pourrait se poser la question de savoir si dans le cas général il existe toujours un point fixe commun à \(f\) et \(g\). Cette conjecture a été émise en 1954. La réponse (négative) a été apportée en 1969. Le lecteur curieux pourra se reporter à la (longue) discussion : http://www.les-mathematiques.net/phorum/read.php?4,546479


Exercice 657 ***

13 janvier 2021 20:49 — Par Alain Soyeur François Capaces Emmanuel Vieillard-Baron
Soit \(f:\mathbb{R} \mapsto \mathbb{R}\) une fonction continue, \(n\in\mathbb{N}^*\) et \(x_1,\dots,x_n \in \mathbb{R}\). Montrer qu’il existe \(c\in \mathbb{R}\) tel que \[f(c)= \dfrac{f(x_1)+\dots +f(x_n)}{n}\]
( ).
Résoudre d’abord l’exercice pour \(n=2\) en faisant un dessin. Passer ensuite au cas général.


[ID: 652] [Date de publication: 13 janvier 2021 20:49] [Catégorie(s): Théorème des valeurs intermédiaires ] [ Nombre commentaires: 0] [nombre d'éditeurs: 1 ] [Editeur(s): Emmanuel Vieillard-Baron ] [nombre d'auteurs: 3 ] [Auteur(s): Alain Soyeur François Capaces Emmanuel Vieillard-Baron ]
Accordéon
Titre
Solution
Texte

Exercice 657
Par Alain Soyeur François Capaces Emmanuel Vieillard-Baron le 13 janvier 2021 20:49

Notons \[t=\dfrac{f(x_1)+\dots+f(x_n)}{n}\] Montrons qu’il existe \(i\in \llbracket 1,n\rrbracket\) tel que \(t\leqslant f(x_i)\). Par l’absurde, si \(\forall k\in [1,n], f(x_k)<t\), en additionnant ces inégalités, on aurait \[nt=\sum_{k=1}^n f(x_k) < nt\] ce qui est absurde. On montre de même qu’il existe \(j\in [1,n]\) tel que \(t\geqslant f(x_j)\).

Par conséquent, \(t\in [f(x_j),f(x_i)]\) et d’après le théorème des valeurs intermédiaires, comme \(f\) est continue sur \(\mathbb{R}\), il existe \(c\in [x_j,x_i]\) tel que \(t=f(c)\).


Exercice 959 ***

13 janvier 2021 20:49 — Par Alain Soyeur François Capaces Emmanuel Vieillard-Baron

Soit une fonction \(f : [0, +\infty[\mapsto \mathbb{R}\) continue telle que \(\forall x \geqslant 0\), \(f(x) \geqslant 0\). On suppose que \(\dfrac{f(x)}{x} \xrightarrow[x \rightarrow +\infty]{} l\) avec \(0 < l < 1\). Montrez que la fonction \(f\) admet un point fixe.



[ID: 654] [Date de publication: 13 janvier 2021 20:49] [Catégorie(s): Théorème des valeurs intermédiaires ] [ Nombre commentaires: 0] [nombre d'éditeurs: 1 ] [Editeur(s): Emmanuel Vieillard-Baron ] [nombre d'auteurs: 3 ] [Auteur(s): Alain Soyeur François Capaces Emmanuel Vieillard-Baron ]
Accordéon
Titre
Solution
Texte

Exercice 959
Par Alain Soyeur François Capaces Emmanuel Vieillard-Baron le 13 janvier 2021 20:49

Considérons un réel \(k \in\mathbb{R}\) tel que \(l < k < 1\). Comme \(\dfrac{f(x)}{x}\xrightarrow[x \rightarrow +\infty]{} l\), il existe \(A > 0\) tel que \(\forall x \geqslant A\), \(\dfrac{f(x)}{x} \leqslant k\). Donc pour \(x \geqslant A\), \(f(x) \leqslant kx\). Considérons la fonction \(\varphi\) définie sur \([0, +\infty[\) par \(\varphi(x) = f(x) - x\). On a \(\varphi(0) = f(0) \geqslant 0\), et puisque \(\varphi(x) \leqslant(k-1) x\) pour \(x \geqslant A\), et que \((k-1) < 0\), \(\varphi(x) \xrightarrow[x \rightarrow +\infty]{} -\infty\). Donc il existe \(B > A\) tel que \(\varphi(B) < 0\). En utilisant le théorème des valeurs intermédiaires entre \(0\) et \(B\), on montre l’existence d’un zéro de la fonction \(\varphi\) et donc d’un point fixe de la fonction \(f\).


Accordéon
Titre
Solution
Texte

Exercice 60
Par Alain Soyeur François Capaces Emmanuel Vieillard-Baron le 13 janvier 2021 20:49
  1. Utilisons la minoration de l’inégalité triangulaire et le fait que \(f\) est contractante : pour tout \(x \in \mathbb{R}\), \[\lvert f(x) \rvert - \lvert f(0) \rvert \leqslant k \lvert x-0 \rvert\]

  2. En utilisant la majoration précédente, pour \(x > 0\), on obtient que : \[g(x) \leqslant\lvert f(0) \rvert + (k-1)x \xrightarrow[x \rightarrow +\infty]{} -\infty\] et pour \(x < 0\), \[g(x) \geqslant-\lvert f(0) \rvert + (k-1)x \xrightarrow[x \rightarrow -\infty]{} +\infty\] On conclut alors grâce au théorème des gendarmes.

  3. Comme \(g(x)\xrightarrow[x \rightarrow +\infty]{} -\infty\) et que \(g(x)\xrightarrow[x \rightarrow -\infty]{} +\infty\), il existe \(A,B\in\mathbb{R}\) tels que \(g\left(A\right)< 0\) et \(g\left(B\right)>0\). On applique alors le théorème des valeurs intermédiaires à \(g\) sur le segment \(\left[A,B\right]\) et on montre qu’il existe \(x_0 \in \mathbb{R}\) tel que \(g(x_0) = 0\). Le réel \(x_0\) est un point fixe de \(f\). Il est unique car si \(x_0'\) est un autre point fixe de \(f\) alors comme \(f\) est une fonction contractante, il vient que \[\left|x_0-x_0'\right|=\left|f\left(x_0\right)-f\left(x_0'\right)\right|\leqslant k\left|x_0 - x_0'\right|\] ce qui est impossible, à moins que \(x_0=x_0'\), car \(k\in\left[0,1\right[\).


;
Success message!